Tài liệu Bất đẳng thức thuần nhất P1 docx

30 733 6
Tài liệu Bất đẳng thức thuần nhất P1 docx

Đang tải... (xem toàn văn)

Tài liệu hạn chế xem trước, để xem đầy đủ mời bạn chọn Tải xuống

Thông tin tài liệu

Chương 4 Bất đẳng thức dạng thuần nhất bậc Tính thuần nhất bậc (đồng bậc, thuần nhất) là một tiêu chuẩn đầu tiên phải tính đến khi so sánh các đại lượng. Các bất đẳng thức cổ điển ta đã biết như bất đẳng thức giữa trung các đạ i lượng trung bình, Cauchy, H ¨ older, Minkowski, Chebychev, . . . , đều là các bất đẳng thức dạng đồng bậc. 1 Trong chương này, chúng ta sẽ đề cập tới các phương pháp cơ bản để chứng minh bất đẳng thức đồng bậc, cũng như cách chuyển từ một bất đẳng thức không đồng bậc về một bất đẳng thức đồng bậc. Nắm vững và vận dụng nhuần nhuyễn các phương pháp này, chúng ta có thể chứng minh được nhiều lớp bất đẳng thức sơ cấp. 4.1 Bất đẳng thức dạng thuần nhất bậc Hàm số f (x 1 , x 2 , . . . , x n ) của các b iế n số thực x 1 , x 2 , . . . , x n được là hàm thuần nhất bậc m nếu với mọi số thực t ta có f (tx 1 , tx 2 , . . . , tx n ) = t m f (x 1 , x 2 , . . . , x n ), với t ∈ R − {0}, và x i ∈ R, i = 1, 2, . . . , n, m, n ∈ N, m = 0, n ≥ 2. Số tự nhiên m được gọi là bậc của đa thức đồng bậc. Bất đẳng thức dạng f (x 1 , x 2 , . . . , x n ) ≥ 0, với f là một hàm thuần nhất được gọi là bất đẳng thức thuần nhất (bậc m). Khái niệm bất đẳng thức đồng bậ c liên quan chặt chẽ với đa thức đồng bậc. Thí dụ, hai đa thức sau là hai đa thức đồng 1 Đây là một chương trong cuốn sách Bất đẳng thức, Suy luận & Khám phá đã xuất bản của tác giả Phạm Văn Thuận, Lê Vĩ. 119 pvthuan 4.2. Đồng bậc hoá bất đẳng thức 120 bậc đồng bậc g(x) = x 5 + y 5 + 8x 2 y 3 , f (x) = x 2 y + 4yx 2 −3x 3 + 10y 3 . Từng đơn thức trong đa t hức thứ nhất có bậc là năm, còn mỗi đơn thức trong đa thức thứ hai có bậc là ba. Cũng cần chú ý rằng đa thức kiểu như f (x) = (x + 2y) 3 + 101x 2 không phải là đồng bậc. 4.2 Đồng bậc hoá bất đẳng thức Với những bất đẳ ng thức có điều kiện, ta có t hể chuyển về dạng bất đẳng thức đồng bậc. Điều kiện cho trước thường là một hệ thức liên hệ giữa các biến số. Từ giả thiết đã cho ta có thể viết bất đẳng thức cần chứng minh dưới dạng đồng bậc. Bài toán 4.1. Cho các số thực không âm a, b thoả mãn điều kiện a + b = 2, chứng minh dãy bất đẳng thức 2 ≤ a 2 + b 2 ≤ a 3 + b 3 ≤ a 4 + b 4 . Chứng minh. Ta lần lượt chứng minh từng bất đ ẳng thức. Mỗi vế bấ t đẳng thức hơn kém nhau một bậc; mà ta cũng thấy rằng biểu thức ở điều kiện cho trước có dạng bậc nhất. Sử dụng giả thiết này ta làm cân bằng bậc của các bất đẳng thức. Trước hết ta chứng minh 2 ≤ a 2 + b 2 . Thật vậy, nhân hai vế với hai, và viết nó dưới dạng tương đương (a + b) 2 ≤ 2(a 2 + b 2 ). Dễ dàng quy bất đẳng thức này về dạng (a − b) 2 ≥ 0. Đối với bất đ ẳng thức thứ hai, ta viết lại dưới dạng (a 2 + b 2 )(a + b) ≤ 2(a 3 + b 3 ). Bất đẳng thức này tương đương với a 3 + b 3 ≥ ab 2 + a 2 b, hay (a −b) 2 (a + b) ≥ 0. Điều này hiển nhiên đúng với a, b > 0. Bất đẳng thức cuối cùng làm tương tự. Bài toán 4.2. Cho các số thực a, b, c thoả mãn điều kiện a 2/3 + b 2/3 + c 2/3 = 3, chứng minh bất đẳng thức a 2 + b 2 + c 2 ≥ a 4/3 + b 4/3 + c 4/3 . Hỏi dấu đẳng thức xảy ra khi nào? pvthuan 4.3. Chuẩn hoá bất đẳng thức 121 Chứng minh. Để bỏ số mũ dạng hữu tỉ, ta đưa ra biến mới như sau. Đặt a 1/3 = x, b 1/3 = y, c 1/3 = z. Khi đó, ta cần chứng minh bất đẳng thức x 6 + y 6 + z 6 ≥ x 4 + y 4 + z 4 với điều kiện x 2 + y 2 + z 2 = 3. Sử dụng giả thiết ta viết được bất đẳng thức cần chứng minh dưới dạng tương đương là 3(x 6 + y 6 + z 6 ) ≥ (x 2 + y 2 + z 2 )(x 4 + y 4 + z 4 ). Nhân khai triển và nhóm các số hạng cho ta (x 2 − y 2 ) 2 (x 2 + y 2 ) + (y 2 − z 2 ) 2 (y 2 + z 2 ) + (z 2 − x 2 ) 2 (z 2 + x 2 ) ≥ 0. Bất đẳng thức này hiển nhiên đúng. 4.3 Chuẩn hoá bấ t đẳng thức Xét bất đẳng thức đồng b ậc dạng f (x 1 , x 2 , . . . , x n ) ≥ g(x 1 , x 2 , . . . , x n ), trong đó f và g là hai đa thức đồng bậ c . Do tính chất của hàm thuần nhất, ta có thể chuyển việc chứng minh bất đẳng thức trên về việc chứng minh bất đẳng thức f (x 1 , x 2 , . . . , x n ) ≥ λ với mọi x 1 , x 2 , . . . , x n thỏa mãn điều kiện g(x 1 , x 2 , . . . , x n ) = λ . Chuẩn hóa một cách thích hợp, ta có thể làm đơn giản các biểu thức của bất đẳng thức cần chứng minh, tận dụng được một số tính chất đặc biệt của các hằng số. Nếu biết quan sát và lựa chọn những điều kiện thích hợp, nghĩa là lúc ấy như có thêm giả thiết, ta sẽ có lời giải gọn gàng, sáng sủa. Trong mục này chúng tôi đưa ra một số cách chọn điều kiện kiểu như vậy. Bài toán 4.3. Chứng minh rằng nếu a, b, c > 0 thì (a + b −c) 2 c 2 + (b + a) 2 + (a + c −b) 2 b 2 + (c + a) 2 + (c + b − a) 2 a 2 + (b + c) 2  3 5 . Chứng minh. Bậc của cả hai vế là không. Đặt x = a/(a + b + c), y = b/(a + b + c), z = c/(a + b + c), thế thì x + y + z = 1. Do đó, ta viết bất đẳng thức cần chứng minh dưới dạng (4.1) (1 − 2z) 2 z 2 + (1 − z) 2 + (1 −2y) 2 y 2 + (1 − y) 2 + (1 − 2x) 2 x 2 + (1 − x) 2  3 5 . Chú ý rằng (1 −2t) 2 t 2 + (1 −t) 2 = 4t 2 −4t + 1 2t 2 −2t + 1 = 2(2t 2 − 2t + 1) − 1 2t 2 −2t + 1 = 2 − 1 2t 2 −2t + 1 . pvthuan 4.3. Chuẩn hoá bất đẳng thức 122 Từ đó, ta có thể viết lại bất đẳng thức ( 4.1) dưới dạng 1 2x 2 −2x + 1 + 1 2y 2 −2y + 1 + 1 2z 2 −2z + 1 ≤ 27 5 Ta cần tìm số δ sao cho với 0 < t < 1 thì 1 2t 2 − 2t + 1 ≤ 9 5 + δ  t − 1 3  . Chuyển 9 5 sang vế trái và quy đồng cho ta (4.2) −18t 2 + 18t −4 2t 2 −2t + 1 − δ  t − 1 3  ≤ 0. Để ý rằng t − 1 3 là nhân tử chung của vế trái c ủa (4.2) nên ta viết lại bất đẳng thức đó dưới dạng tương đương (4.3)  t − 1 3   12 − 18t 2t 2 −2t + 1 − δ  ≤ 0. Bây giờ ta cần tìm δ sao cho biểu thức trong ngoặc thứ hai của (4.3) nhận t − 1 3 làm nhân tử. Thế thì ta thay t = 1 3 vào biểu thức δ = 12−18t 2t 2 −2t+1 sẽ thu được δ = 54 5 . Với giá trị này của δ , ta viết lại (4.3) dưới dạng  t − 1 3   12 − 18t 2t 2 −2t + 1 − 54 5  ≤ 0. Bất đẳng thức này tương đương với −(t − 1 3 ) 2 (18t + 1 3 ) ≤ 0. Điều này hiển nhiên đúng. Vậy, ta có đánh giá sau đây 1 2t 2 −2t + 1 ≤ 9 5 + 54 5  t − 1 3  . Sử dụng ước lượng này ba lần cho x, y, z ta sẽ có ngay điều phải chứng minh. Phép chứng minh hoàn tất. Nói chung, những bài bất đẳng thức có một vế là tổng của ba phân thức như trên là rất khó hoặc không thể đánh giá được từng phân thức. Cách chọn trên cho phép ta làm được điều khó khăn này một cách dễ dàng dựa tr ên tính chất cơ bản về bất đ ẳng thức, phân số, và tam thức bậc hai. Tổng quát hơn, ta có thể chứng minh được nhiều b ất đẳng thức đối xứng đồng bậc bằng cách đặt x = ka/(a + b + c) và tương tự với y, z ta được x + y + z = k, mà không làm mất tính đối xứng của bất đẳng thức ban đầu. Một cách tương pvthuan 4.3. Chuẩn hoá bất đẳng thức 123 tự, ta có thể giải bài toán sau đây và thiết lập được nhiều bài bất đẳng thức chứa các biểu thức phân thức kiểu như bài toán trong đề thi vô địch Hoa Kỳ năm 2003 (2a + b + c) 2 2a 2 + (b + c) 2 + (2b + c + a) 2 2b 2 + (c + a) 2 + (2c + a + b) 2 2c 2 + (a + b) 2  8, với giả thiết a, b, c là các số dương. Ta thấy rằng tử t hức và mẫu thức của mỗi phân thức ở vế trái đều là các đa thức bậc hai. Vì thế vế trái của bất đẳng thức có bậc là không. Không mất tổng quát, ta c ó thể giả sử rằng a + b + c = 1. Và từ đó tiếp tục như trên. Ta xét thêm một bất đẳng thức khác có dạng “tương tự" như trên, ấy là chứng minh rằng (4a + b −c) 2 2a 2 + (b + c) 2 + (4b + c − a) 2 2b 2 + (c + a) 2 + (4c + a −b) 2 2c 2 + (a + b) 2  8. Nhưng thật đáng tiếc rằng kỹ thuật trên đây lại không có tác dụng với bài toán này. Điều đó là do ta không thể cô lập hay đưa mỗi phân thức về dạ ng một biến số thông qua điều kiện tổng ba số bằng k. Đây cũng là ý tưởng cơ sở quan trọng của cách làm này. Bài toán 4.4. Chứng minh rằng nếu a, b, c > 0 thì (a + b + c) 2 a 2 + b 2 + c 2 + 1 2  a 3 + b 3 + c 3 abc − a 2 + b 2 + c 2 ab + bc + ca   4. Chứng minh. Bài này là tổng của ba bất đẳng thức ngược chiều nhau. Biểu thức thứ nhất trong ngoặc đạt giá trị nhỏ nhất; còn hai biểu thức kia đạt giá trị lớn nhất. Không giảm tổng quát ta có thể giả sử ω = a 2 + b 2 + c 2 , chọn ω = 3, và sử dụng các hằng đẳng thức (a + b + c) 2 = a 2 + b 2 + c 2 + 2(ab + bc + ca), a 3 + b 3 + c 3 − 3abc = (a + b + c)(a 2 + b 2 + c 2 − ab −bc − ca). Do đó, ta viết vế trái bất đẳng thức dưới dạ ng F = 5 2 + 2 λ 3 + 1 2 η (3 − λ ) − 3 2 λ , trong đó λ = ab + bc + ca, và η = 1/(ab) + 1/(bc) + 1/(ca), ta đã biết rằng λ  3, mặt khác sử dụng bất đẳng thức quen biết η  9/ λ , F  5 2 + 2 λ 3 + 9 2 λ (3 − λ ) − 3 2 λ = −2 + 2 λ 3 + 12 λ . pvthuan 4.3. Chuẩn hoá bất đẳng thức 124 Do đó, để chứng minh F  4, ta chỉ cần chỉ ra rằng λ /3 + 6/ λ  3. Thật vậy, sử dụng bất đẳng thức giữa trung bình cộng và trung bình nhân sau khi đã tách số hạng sau để có dấu đẳng thức, ta có λ 3 + 3 λ + 3 λ 3   λ 3 . 3 λ . 3 λ  1/3 =  3 λ  1/3  1. Phép chứng minh đã hoàn tất. Bằng cách đặt tương tự, ta có thể thiết lập các biể u thức đối xứng cùng bậc rồi chọn một điều kiện nào đó, ước lượng các giá trị khi các biến số bằng nhau, ta có thể xây dựng được các bất đẳng thức mới. Bài toán 4.5. Chứng minh rằng với mọi số thực a, b, c thì 6(a + b + c)(a 2 + b 2 + c 2 )  27abc + 10(a 2 + b 2 + c 2 ) 3/2 . Chứng minh. Rõ ràng hai vế của bất đẳng thức này đều có dạng bậc ba. Nhưng nếu tiếp tục lựa chọn như thí dụ trên sẽ không hiệu quả nữa bởi biểu thức a 2 + b 2 + c 2 có số mũ dạng hữu tỷ. Nếu cả ba số bằng không t hì bất đẳng thức hiển nhiên đúng. Nếu có một số khác không, ta đặt ω = a 2 + b 2 + c 2 , và giả sử |a|  |b|  |c|, chọn ω = 9 để tránh dạng số vô tỷ. Thế thì bất đẳng thức trên có dạng 2(a + b + c) − abc  10. Sau đây là một thí dụ cho thấy rằng phép chọn điều kiện thích hợp có thể cho ta lời giải ngắn gọn, độc đáo. Bài toán 4.6. Giả sử a, b, c là các số thực dương, chứng minh bất đẳ ng thức 3  (a + b)(b + c)(c + a) 8 ≥  ab + bc + ca 3 . Chứng minh. Chọn ab + bc + ca = 3, thành thử là bây giờ ta chỉ cần chứng minh rằng (a + b)(b + c)(c + a) ≥ 8. Nhưng chú ý rằng (a + b)(b + c)(c + a) = (a + b + c)(ab + bc + ca) − abc. Phép chứng minh hoàn tất nếu ta chỉ ra được rằng 3(a + b + c) − abc ≥ 8. Theo bất đẳng thức giữa trung bì nh cộng và trung bình nhân ta có −abc ≥ −1, và (a + b + c) 2 ≥ 3(ab + bc + ca). Từ đ ó suy ra điều phải chứng minh. Có hai câu hỏi cần đặt ra sau lời giải này là tại sao lại chọn số 3, và chọn như thế có lợi ích gì. Cái tưởng chừng như khó nhất của bài toán là sự có mặt của hai căn thức khác nhau đã được vượt qua dễ dàng. Bài toán 4.7 . Cho các số thực dương x, y, z và đặt x + y + z = p, xy + yz + zx = q, và xyz = r. Giả sử p = 1, hãy biểu diễn các biểu thức x 2 + y 2 + z 2 , theo p, q, và biểu diễn x 3 + y 3 + z 3 , x 4 + y 4 + z 4 theo p, q, r. pvthuan 4.3. Chuẩn hoá bất đẳng thức 125 Từ kết quả của bài toán trên đây, người ta có thể áp đặt điều kiện để x ây dựng các bất đẳng thức đối xứng ba biến có điều kiện. Với một bài toán bất đẳng thức đồng bậc ba biến, ta có thể lựa chọn tuỳ ý một (chỉ một thôi) trong ba điều kiện p = 1, q = 1, r = 1. Bài toán 4.8. Xét ba số thực không âm a, b, c thỏa mãn a + b + c = 1, chứng minh rằng a √ a + b 2 + b √ b + c 2 + c √ c + a 2 ≤ 3 2 . Chứng minh. Lời giải sau đây của Võ Quốc Bá Cẩn. Giả sử L là vế trái của bất đẳng thức. Sử dụng bất đẳng thức Cauchy Schwarz, ta c ó L 2 ≤ (a + b + c)  a a + b 2 + b b + c 2 + c c + a 2  = a a + b 2 + b b + c 2 + c c + a 2 . Do đó, ta chỉ cần chứng minh rằng a a + b 2 + b b + c 2 + c c + a 2 ≤ 9 4 . Bất đẳng thức này lại có thể viết dưới dạng tương đương a 2 b + a 2 + b 2 c + b 2 + c 2 a + c 2 ≥ 3 4 . Viết bất đẳng thức này dưới dạng đồng bậc. Lại áp dụng bất đẳng thức Cauchy Schwarz dạng Engel, rồi cuối cùng quy về việc chứng minh a 4 + b 4 + c 4 + 5(a 2 b 2 + b 2 c 2 + c 2 a 2 ) ≥ 3(ab 3 + bc 3 + ca 3 ) + 3abc(a + b + c). Theo bất đẳng thức giữa trung bình cộng và trung bình nhân thì a 2 b 2 + b 2 c 2 + c 2 a 2 ≥ abc(a + b + c). Và thế nên ta cần chứng minh rằng a 4 + b 4 + c 4 + 2(a 2 b 2 + b 2 c 2 + c 2 a 2 ) ≥ 3(ab 3 + bc 3 + ca 3 ). Bất đẳng thức này lại viết được về dạng ∑ cyclic (a 2 −c 2 −2ab + bc + ca) 2 ≥ 0. Phép chứng minh hoàn tất. pvthuan 4.4. Lớp hàm đối xứng sơ cấp ba biến 126 Phần này làm bất đẳng thức đồng bậc trở nên không đồng bậc bằng cách lựa chọn được những điều kiên thích hợp; có lợi cho biến đổi và tính toán. Bài toán 4.9. Cho ba số thực dương a, b, c sao cho a + b + c = 1, chứng minh bất đẳng thức 9(a 3 + b 3 + c 3 ) −10(a 5 + b 5 + c 5 ) ≥ 1. Bài toán 4.10. Chứng minh rằng nếu a, b, c > 0 thì ta có bất đẳng thức (a + b + c) 2 a 2 + b 2 + c 2 + 1 2  a 2 + b 2 + c 2 ab + bc + ca − a 3 + b 3 + c 3 abc  ≤ 2. 4.4 Lớp hàm đối x ứng sơ cấp ba biến Tất cả các bất đẳ ng thức đối xứng ba biến số đều có thể quy về các hà m đối xứng cơ bản của p = x + y + z, q = xy + yz + zx, và r = xyz. Trong tiết này ta sẽ lần lượt xét các bài toán bấ t đẳ ng thức, từ dễ đến khó, có thể giải theo đường lối này. Sau khi đã viết bất đẳng thức cần chứng minh theo p, q, r, ta chỉ cần khảo sát bất đẳng thức này theo ba biến mới p, q, r. Điểm mạnh nhất của phương pháp này là xử lý được những bất đẳng thức đối xứng ba biến, chặt và khó, vì ta không thực hiện nhiều phép ước lượng trung gian thô và điều đó cũng có nghĩa là ta p hải làm việc với nhiều bước tính toán nhất là trong bài toán dạng phân thức, hoặc bậc cao. Trước tiên, bạn đọc hãy tự kiểm tra các kết quả cơ bản sau đây. x 2 + y 2 + z 2 = p 2 −2q, x 3 + y 3 + z 3 = p(p 2 − 3q) + 3r, x 4 + y 4 + z 4 = (p 2 − 2q) 2 − 2(q 2 −2pr). Một vấn đề nữa cần đặt ra là thứ tự so sánh giữa các bộ p, q, r như thế nào. Một nguyên tắc là phải đảm bảo được tính đồng bậc. Với ba số a, b, c là nghiệm của một phương trình bậc ba f (x) = x 3 + px 2 + qx + r thì ta có thể viết f (x) = (x − a)(x − b)(x −c). Khai triển đa thức ta được (x 2 − xb − xa + ab)(x − c) = x 3 − x 2 (a + b + c) + x(ab + bc + ca) − abc. Bây giờ sử dụng đồng nhất thức ta được      a + b + c = −p, ab + bc + ca = q, abc = −r. Dựa vào đặc điểm trên đây, ta có thể phát biểu nhiều bài toán bất đẳng thức với giả thiết liên quan đ ến nghiệm của một phương trình bậc ba. Trước hết, ta hãy quan sát các bất đẳng thức liên quan đến p, q, r. Đó là bất đẳng thức Schur. pvthuan 4.4. Lớp hàm đối xứng sơ cấp ba biến 127 Định lý 4.1 (I. Schur). Nếu x, y, z là các số thực dương và t là một số thực, thì x t (x − y)(x − z) + y t (y − z)(y − x) + z t (z − y)(z − x) ≥ 0. Chứng minh. Vì bất đẳng thức cần chứng minh có dạng đối xứng nên ta có thể giả sử rằng x ≥ y ≥ z. Viết lại bất đẳng thức đã cho bằng cách nhóm nhân tử chung, ta được (x − y){x t (x − z) − y t (y −z)} + z t (x − z)(y −z) ≥ 0. Dễ thấy rằng bất đẳng thức này đúng. Có một số trường hợp riêng của bất đẳng thức này rất đáng chú ý là trường hợp t = 1, 2, 3. Lấy các giá trị này của t và khai triển đa thức ta sẽ thu được các trường hợp lý thú và hữu ích. Bài toán 4.11. Cho các số thực dương x, y, z. Đặt x + y + z = p, xy + yz + zx = q và xyz = r, chứng minh các bất đ ẳng thức p 3 −4pq + 9r ≥ 0, p 4 − 5p 2 q + 4q 2 + 6pr ≥ 0, pq −9r ≥ 0. Chứng minh. Ba bất đẳng thức trê n có thể viết lại dưới dạng x(x − y)(x − z) + y(y −z)(y − x) + z(z − x)(z − y) ≥ 0,(4.4) x 2 (x − y)(x − z) + y 2 (y −z)(y −x) + z 2 (z − x)(z − y) ≥ 0,(4.5) x(y − z) 2 + y(z − x) 2 + z(x − y) 2 ≥ 0.(4.6) Dễ thấy hai bất đẳng thức ba đầu là trường hợp t = 1, và t = 2 của bất đ ẳng thức Schur. Bất đẳng t hức cuối cùng là hiển nhiên. Bài toán 4.12. Cho các số thực dương x, y, z. Đặt x + y + z = p, xy + yz + zx = q và xyz = r, chứng minh các bất đ ẳng thức p 2 ≥ 3q p 3 ≥ 27r q 2 ≥ 3pr 2p 3 + 9r ≥ 7pq p 2 q + 3pr ≥ 4q 2 p 2 q ≥ 3pr + 4p 2 q p 4 + 3q 2 ≥ 4p 2 q pq 2 + 3q 2 ≥ 4p 2 q pq 2 ≥ 2p 2 r + 3qr 2p 3 + 9r 2 ≥ 7pqr q 3 + 9r 2 ≥ 4pqr p 3 q + q 3 ≥ 6pqr Từ các kết quả cơ bản trên ta có thể phát biểu b ài toán dưới dạng nghiệm của một phương trình bậc ba. Chẳng hạn, ta xét bài toán sau đây. Bài toán 4.13. Xét ba số thực a, b, c sao cho đ a thức x 3 + ax 2 + bx + c có ba nghiệm. Chứng minh rằng 12ab + 27c ≤ 6a 3 + 10(a 2 −2b) 3/2 . Hỏi dấu đẳng thức xảy ra khi nào? pvthuan 4.4. Lớp hàm đối xứng sơ cấp ba biến 128 Bài toán 4.14. Chứng minh rằng nếu x, y, z > 0 thì (xy + yz + zx)  1 (x + y) 2 + 1 (y + z) 2 + 1 (x + z) 2  ≥ 9 4 . Chứng minh. Lời giải của Hojoo Lee. Sử dụng phép thay thế p, q, r như trê n. Chú ý tính chất (x + y)(y + z)(z + x) = (x + y + z)(xy + yz + zx) − xyz = pq −r. Ta có thể chuyển bất đẳng thức cần chứng minh về dạng sau theo p, q, r q  (p 2 + q) 2 − 4p(pq −r) (pq − r) 2  ≥ 9 4 . Biến đổi tương đương và tính toán cho ta 4p 4 q − 17p 2 q 2 + 4q 3 + 34pqr −9r 2 ≥ 0 pq(p 3 −4pqr + 9r) + q(p 4 − 5p 2 q + 4q 2 + 6pr) + r(pq −9r) ≥ 0 Dễ thấy rằng bất đẳng thức cuối cùng đúng theo bài toán trên. Bài toán 4.15. Cho ba số thực dương x, y, z thỏa mãn điều kiện x + y + z = 1, chứng minh rằng (1 − x 2 ) 2 + (1 − y 2 ) 2 + (1 − z 2 ) 2 ≤ (1 + x)(1 + y)(1 + z). Chứng minh. Đây là bất đẳng thức đối xứng, vai trò giữa các biến hoàn toàn bình đẳng. Theo cách đặt trên, ta có p = x + y + z, q = xy + yz + zx, và r = xyz. Thế thì bất đẳng thức cần chứng minh có dạng 3 −2(p 2 − 2q) + (p 2 −2q) 2 −2(q 2 −2pr) ≤ 1 + p + q + r. Chú ý rằng p = 1 nên bất đẳng thức trên lại có dạng 3 − 2(1 − 2q) + (1 − 2q) 2 −2(q 2 −2r) ≤ 1 + p + q + r, hay 2q 2 −q + 3r ≤ 0. Vì pq ≥ 9r nên q ≥ 9r. Do đó, phép c hứng minh hoàn tất nếu ta có 2q 2 −q + 1 3 q ≤ 0, điều này tương đương với 2q 2 − 2 3 q ≤ 0. Dễ thấy bất đẳng thức này đúng với 0 ≤ q ≤ 1/3. Điều này đúng vì ta có p 2 ≥ 3q. Bài toán 4.16. Xét ba số thực dương x, y, z thỏa mãn điều kiện xyz = 1, chứng minh rằng 1 (1 + x) 2 + 1 (1 + y) 2 + 1 (1 + z) 2 + 2 (1 + x)(1 + y)(1 + z) ≥ 1. pvthuan [...]... ca) ≥ 8 27 hu Chứng minh Bất đẳng thức bến trái hiển nhiên Ta sẽ chứng minh bất đẳng thức bên phải Đặt p = a + b + c, q = ab + bc + ca, r = abc Thế thì bất đẳng thức cần chứng minh sẽ có dạng 1 − q + pr − r2 ≥ pv t (4.7) 8 27 Theo bất đẳng thức I Schur, p3 − 4pq + 9r ≥ 0, và chú ý rằng nhờ giả thiết ta có 4q − p2 = 2q − 1 Do đó ta có quan hệ 9r ≥ p(2q − 1) Theo bất đẳng thức giữa trung bình cộng... biến Bất đẳng thức cần chứng minh tương đương với mỗi bất đẳng thức (1 − q)2 (1 − q) − (1 − q)(q + 3r) − q2 + 5r2 ≥ 2[(1 − q)q2 − r] q3 + q2 − 4q + 3qr + 4r + 1 ≥ 0, hay là (4.13) 27q3 + 27q2 − 108q + 27r(3q + 4) + 27 ≥ 0 Theo bất đẳng thức Schur, ta có 9r ≥ 4q − 1 Do đó, để chứng minh bất đẳng thức (4.13) ta chứng minh 27q3 + 27q2 − 108q + 3(4q − 1)(3q + 4) + 27 ≥ 0 an Đặt 3q = t thì bất đẳng thức. .. thiết cho ta q = 1, bất đẳng thức cần chứng minh có thể viết theo p, q, r như sau an 1 + p2 1 − ≥ 2 p−r p Bất đẳng thức này có thể viết dưới dạng f ( p) = p3 − 2p2 + r(1 + 2p) ≥ 0 Nếu p ≥ 2 thì bất đẳng thức f ( p) ≥ 0 hiển nhiên đúng Bây giờ ta chỉ cần xét √ 3 ≤ p ≤ 2 Theo bất đẳng thức I Schur, ta có r ≥ Do đó, 4p − p3 (1 + 2p) 9 hu f ( p) ≥ p 3 − 2p2 + 4p − p 3 9 Vế phải bất đẳng thức trên lại viết... (8r − 1)(64r + 10) ≤ 0 Bất đẳng thức này đúng với r ≤ 1/8 Bài toán 4.20 Xét các số thực không âm a, b, c, chứng minh bất đẳng thức an (8a2 + bc)(8b2 + ca)(8c2 + ab) ≤ ( a + b + c)6 Hỏi đẳng thức đạt được khi nào? (4.10) hu Chứng minh Đặt p = a + b + c, q = ab + bc + ca, r = abc Vì bất đẳng thức cần chứng minh có dạng đồng bậc nên ta có thể chọn p = 1 Khi đó, ta viết bất đẳng thức trên dưới dạng sau... − xy 8 Chứng minh Bất đẳng thức bên vế trái là hiển nhiên Ta sẽ chứng minh bất đẳng thức bên phải Ta đặt p = x + y + z, q = xy + yz + zx, và r = xyz Chú ý các đẳng thức sau đây x2 + y2 + z2 = p2 − 2q, x3 + y3 + z3 = p( p2 − 3q) + 3r Bất đẳng thức cần chứng minh sẽ có dạng (theo p, q, r) p − ( p2 − 2q) + p( p2 − 3q) + 3r + r( p2 − 2q) ≤ 9 (1 − q − pr − r2 ) 8 Vì p = 1 nên bất đẳng thức trên lại thu... đây suy ra bất đẳng thức cần chứng minh Phép chứng minh hoàn tất Đẳng thức xảy ra khi và chỉ khi a = b = c = 1 2 (2) Phần tiếp theo ta cũng làm tương tự Sử dụng các đồng nhất thức (8ab + 1)(8bc + 1) + (8bc + 1)(8ca+1) + (8ca + 1)(8ab + 1) = 64pr + 16q + 3 (8ab + 1)(8bc + 1)(8ca + 1) = 512r2 + 64pr + 8q + 1 Bất đẳng thức cần chứng minh có dạng 8q + 2 ≥ 512r2 , vì q + 2r = 1 nên bất đẳng thức trên đây... − ab = 1 − c − q Bất đẳng thức cần chứng minh sẽ được viết dưới dạng ∑ (1 − a − q)(1 − b − q) ≥ 9(1 − a − q)(1 − b − q)(1 − c − q) cyclic Nhân phá ngoặc, và để dễ tính toán ta đặt m = 1 − q, ta sẽ được 3m2 − 3m − 5 ≥ 9m3 − 9m2 + 9mq − 9r Bất đẳng thức này lại viết được dưới dạng 9q3 + 6q2 − 3q + 9r + 1 ≥ 0, tức là q(3q + 1)2 + 9r + 1 − 4q ≥ 0 Bất đẳng thức này đúng vì theo bất đẳng thức Schur, ta có... 1 , ta có bất đẳng thức (4.16) tương đương với 4 q ≥ 12q2 (1 − 3q) + 12r(3q2 + 6q − 2) − 72r2 pv t Bất đẳng thức này đúng bởi vì 12q(1 − 3q) = 4.3q(1 − 3q) ≤ [3q + (1 − 3q)]2 = 1 Suy ra 12q2 (1 − 3q) ≤ q và 0 ≤ q ≤ 1 nên 3q2 + 6q − 2 < 0 Phép chứng minh 4 hoàn tất Đẳng thức xảy ra tại ba điểm có toạ độ khác nhau Bạn đọc tự kiểm tra điều này Bài toán 4.28 Tìm số a nhỏ nhất sao cho bất đẳng thức sau... y + z = 1, chứng minh bất đẳng thức an 1 1 1 27 + + ≤ 1 − xy 1 − yz 1 − zx 8 Chứng minh Ta đặt p = x + y + z, q = xy + yz + zx và r = xyz Sử dụng các đồng nhất thức (1 − xy)(1 − yz)(1 − zx) = 1 − q + pr − r2 (1 − xy)(1 − yz) + (1 − yz)(1 − zx) + (1 − zx)(1 − xy) = 3 − 2q + pr hu Thành thử, bất đẳng thức cần chứng minh có dạng 3 − 2q + pr 27 ≤ 2 8 1 − q + pr − r pv t Bất đẳng thức này tương đương với... (r) đồng biến, suy ra f (r) ≤ f (q2 /3) Lại chú ý rằng f ( q2 /3 ) = q4 − 16 3 23 2 q + q + q 3 3 Bất đẳng thức f (q2 /3) ≤ 1 tương đương với (3q − 1)(q3 − 5q2 + 6q + 3) ≤ 0, an hay là (3q − 1)(q(q − 2)(q − 3) + 3) ≤ 0 Bất đẳng thức này đúng Phép chứng minh hoàn tất hu Bất đẳng thức sau đây có dấu đẳng thức tại một giá trị biên Thế nhưng phương pháp biểu diễn qua hàm đối xứng sơ cấp vẫn có tác dụng . bậc của đa thức đồng bậc. Bất đẳng thức dạng f (x 1 , x 2 , . . . , x n ) ≥ 0, với f là một hàm thuần nhất được gọi là bất đẳng thức thuần nhất (bậc m) 4 Bất đẳng thức dạng thuần nhất bậc Tính thuần nhất bậc (đồng bậc, thuần nhất) là một tiêu chuẩn đầu tiên phải tính đến khi so sánh các đại lượng. Các bất

Ngày đăng: 21/01/2014, 14:20

Từ khóa liên quan

Tài liệu cùng người dùng

  • Đang cập nhật ...

Tài liệu liên quan